LSAT and Law School Admissions Forum

Get expert LSAT preparation and law school admissions advice from PowerScore Test Preparation.

 Administrator
PowerScore Staff
  • PowerScore Staff
  • Posts: 8917
  • Joined: Feb 02, 2011
|
#47398
Complete Question Explanation
(The complete setup for this game can be found here: lsat/viewtopic.php?t=4635)

The correct answer choice is (C)

As with all complete List questions, simply apply the rules to the answer choices.

Answer choice (A): This answer choice is incorrect because S and T both attend L, which is a violation of the third rule.

Answer choice (B): This answer choice is incorrect because R attends H, a violation of the second rule.

Answer choice (C): This is the correct answer choice.

Answer choice (D): This answer choice is incorrect because Q does not attend the first talk that T attends, a violation of the fourth rule.

Answer choice (E): This answer choice is incorrect because S does not attend the first talk that R attends, a violation of the fifth rule.

Get the most out of your LSAT Prep Plus subscription.

Analyze and track your performance with our Testing and Analytics Package.